Difference between revisions of "2013 AMC 12B Problems/Problem 15"
Susanssluk (talk | contribs) m (→Solution) |
(→Video Solution) |
||
(3 intermediate revisions by 2 users not shown) | |||
Line 8: | Line 8: | ||
==Solution== | ==Solution== | ||
The prime factorization of <math> 2013 </math> is <math> 61\cdot11\cdot3 </math>. To have a factor of <math>61</math> in the numerator and to minimize <math>a_1,</math> <math>a_1</math> must equal <math>61</math>. Now we notice that there can be no prime <math>p</math> which is not a factor of <math>2013</math> such that <math> b_1<p<61,</math> because this prime will not be canceled out in the denominator, and will lead to an extra factor in the numerator. The highest prime less than <math>61</math> is <math>59</math>, so there must be a factor of <math>59</math> in the denominator. It follows that <math>b_1 = 59</math> (to minimize <math>b_1</math> as well), so the answer is <math>|61-59| = \boxed{\textbf{(B) }2}</math>. One possible way to express <math> 2013 </math> with <math>(a_1, b_1) = (61, 59)</math> is <cmath> 2013 = \frac{61!\cdot19!\cdot11!}{59!\cdot20!\cdot10!}. </cmath> | The prime factorization of <math> 2013 </math> is <math> 61\cdot11\cdot3 </math>. To have a factor of <math>61</math> in the numerator and to minimize <math>a_1,</math> <math>a_1</math> must equal <math>61</math>. Now we notice that there can be no prime <math>p</math> which is not a factor of <math>2013</math> such that <math> b_1<p<61,</math> because this prime will not be canceled out in the denominator, and will lead to an extra factor in the numerator. The highest prime less than <math>61</math> is <math>59</math>, so there must be a factor of <math>59</math> in the denominator. It follows that <math>b_1 = 59</math> (to minimize <math>b_1</math> as well), so the answer is <math>|61-59| = \boxed{\textbf{(B) }2}</math>. One possible way to express <math> 2013 </math> with <math>(a_1, b_1) = (61, 59)</math> is <cmath> 2013 = \frac{61!\cdot19!\cdot11!}{59!\cdot20!\cdot10!}. </cmath> | ||
+ | |||
+ | ==Video Solution== | ||
+ | https://youtube.com/FvscTObzpwA | ||
+ | |||
+ | ~IceMatrix | ||
+ | |||
+ | ==Video Solution 2== | ||
+ | https://youtu.be/GdfR_UjhYYQ | ||
+ | |||
+ | ~savannahsolver | ||
== See also == | == See also == |
Latest revision as of 15:28, 25 December 2023
- The following problem is from both the 2013 AMC 12B #15 and 2013 AMC 10B #20, so both problems redirect to this page.
Problem
The number is expressed in the form
where and are positive integers and is as small as possible. What is ?
Solution
The prime factorization of is . To have a factor of in the numerator and to minimize must equal . Now we notice that there can be no prime which is not a factor of such that because this prime will not be canceled out in the denominator, and will lead to an extra factor in the numerator. The highest prime less than is , so there must be a factor of in the denominator. It follows that (to minimize as well), so the answer is . One possible way to express with is
Video Solution
https://youtube.com/FvscTObzpwA
~IceMatrix
Video Solution 2
~savannahsolver
See also
2013 AMC 10B (Problems • Answer Key • Resources) | ||
Preceded by Problem 19 |
Followed by Problem 21 | |
1 • 2 • 3 • 4 • 5 • 6 • 7 • 8 • 9 • 10 • 11 • 12 • 13 • 14 • 15 • 16 • 17 • 18 • 19 • 20 • 21 • 22 • 23 • 24 • 25 | ||
All AMC 10 Problems and Solutions |
2013 AMC 12B (Problems • Answer Key • Resources) | |
Preceded by Problem 14 |
Followed by Problem 16 |
1 • 2 • 3 • 4 • 5 • 6 • 7 • 8 • 9 • 10 • 11 • 12 • 13 • 14 • 15 • 16 • 17 • 18 • 19 • 20 • 21 • 22 • 23 • 24 • 25 | |
All AMC 12 Problems and Solutions |
The problems on this page are copyrighted by the Mathematical Association of America's American Mathematics Competitions.